Administración     

Olimpiadas de Matemáticas
Página de preparación y problemas

OME Local
OME Nacional
OIM
OME Andalucía
Retos UJA
Selector
La base de datos contiene 1154 problemas y 775 soluciones.

XLI Olimpiada Matemática Española (fase local) — 2005

Sesión 1 —  Viernes 21 de enero de 2005 (mañana)

Problema 194
Sean $a,b,c$ reales no nulos con $a\neq b$ y supongamos que los polinomios $x^2+ax+bc$ y $x^2+bx+ac$ tienen una raíz común. Demostrar que las otras dos raíces son raíces del polinomio $x^2+cx+ab$.
pistasolución 1info
Pista. Utiliza las relaciones de Cardano en los polinomios del enunciado.
Solución. Si llamamos $r$ y $s$ a las raíces de $x^2+ax+bc$ y llamamos $r$ y $t$ a las de $x^2+bx+ac$, obtenemos que $rs=bc$ y $rt=ac$. Dividiendo una ecuación entre la otra y como los números $a,b,c,r,s,t$ son todos no nulos (¿por qué?), llegamos a que $as-bt=0$. Por otro lado, tenemos que $r+s=-a$ y $r+t=-b$. Restando estas dos ecuaciones obtenemos que $s-t=b-a$. El sistema de ecuaciones lineales \[\left.\begin{array}{c} as-bt=0\\ s-t=b-a \end{array}\right\}\] con incógnitas $s$ y $t$ tiene una única solución ya que $a\neq b$ y ésta es $s=b$ y $t=a$, de donde $r=c$ y $a+b+c=a+(r+s)=a-a=0$. Por tanto, \[(x-s)(x-t)=x^2-(s+t)x+st=x^2+cx+ab,\] y hemos resuelto el problema.
Si crees que el enunciado contiene un error o imprecisión o bien crees que la información sobre la procedencia del problema es incorrecta, puedes notificarlo usando los siguientes botones:
Informar de error en enunciado Informar de procedencia del problema
Problema 720
Sea $M$ un punto interior del segmento $AB$. Se construyen cuadrados $AMCD$ y $BEHM$ en el mismo lado de $AB$. Sea $N$ el punto de intersección (distinto de $M$) de las circunferencias circunscritas a dichos cuadrados.
  1. Probar que los puntos $B$, $N$ y $C$ están alineados.
  2. Probar que el punto $H$ es el ortocentro del triángulo $ABC$.
pistasolución 1info
Pista. ¿Qué relación hay entre los triángulos $AMH$ y $CMB$?
Solución. Los dos triángulos $AHM$ y $CMB$ son congruentes por ser rectángulos y tener los catetos iguales (son iguales a los lados de los cuadrados). Además, podemos pasar de $AHM$ a $CMB$ mediante una rotación de $90^\circ$ con centro en $M$, luego la recta $AH$ (en rojo discontinuo) es perpendicular a $BC$. Ahora bien, la recta $BC$ también es perpendicular a $AN$ ya que $AC$ es un diámetro de la circunferencia circunscrita al cuadrado $AMCD$. Por tanto $AN$ y $AH$ son la misma recta, es decir, $A$, $H$ y $N$ están alineados. Respondemos ahora a las dos cuestiones:
  1. El ángulo $\angle HNB$ es recto ya que $HB$ es un diámetro de la circunferencia circunscrita al cuadrado $MBEH$. Por tanto, tanto $\angle HNB$ como $\angle HNC$ son rectos, lo que nos dice que $B$, $N$ y $C$ están alineados.
  2. Con lo ya demostrado, tenemos que $AN$ es la altura que pasa por $A$ en el triángulo $ABC$ y es obvio que $CM$ es la altura que pasa por el vértice $C$. Su intersección es $H$ y, por consiguiente, $H$ es el ortocentro de $ABC$.
imagen
Si crees que el enunciado contiene un error o imprecisión o bien crees que la información sobre la procedencia del problema es incorrecta, puedes notificarlo usando los siguientes botones:
Informar de error en enunciado Informar de procedencia del problema
Problema 721
Sean $x,y,z$ números reales positivos.
  1. Si $x+y+z\geq 3$, ¿se verifica necesariamente que $\frac{1}{x}+\frac{1}{y}+\frac{1}{z}\leq 3$?
  2. Si $x+y+z\leq 3$, ¿se verifica necesariamente que $\frac{1}{x}+\frac{1}{y}+\frac{1}{z}\geq 3$?
pistasolución 1info
Pista. Utiliza la desigualdad entre las medias aritmética y armónica.
Solución. La respuesta al apartado (a) es negativa. Por ejemplo, los números $x=1$, $y=\frac{1}{2}$ y $z=\frac{3}{2}$ cumplen $x+y+z=3$, pero $\frac{1}{x}+\frac{1}{y}+\frac{1}{z}=\frac{11}{3}\gt 3$.

La respuesta al apartado (b) es afirmativa. Para verlo, usamos la desigualdad entre las medias aritmética y armónica aplicada a los tres números positivos $x,y,z$: \[\frac{3}{\frac{1}{x}+\frac{1}{y}+\frac{1}{z}}\leq \frac{x+y+z}{3}\leq 1\ \Longleftrightarrow\ \frac{1}{x}+\frac{1}{y}+\frac{1}{z}\geq 3.\]

Si crees que el enunciado contiene un error o imprecisión o bien crees que la información sobre la procedencia del problema es incorrecta, puedes notificarlo usando los siguientes botones:
Informar de error en enunciado Informar de procedencia del problema

Sesión 2 —  Viernes 21 de enero de 2005 (tarde)

Problema 722
Se considera un triángulo $ABC$ con $\angle BAC=45^\circ$ y $\angle ACB = 30^\circ$. Si $M$ es el punto medio del lado $BC$, demostrar que $\angle AMB = 45^\circ$ y que $BC\cdot AC=2\cdot AM\cdot AB$.
pistasolución 1info
Pista. Aplica el teorema del seno a los triángulos $ABM$ y $ABC$ junto con la fórmula para la longitud de la mediana $AM$ en términos de los lados del triángulo.
Solución. Llamemos a los lados del triángulo $a=AB$, $b=AC$ u $c=AB$, como es usual, y a la mediana $m=AM$. El teorema del seno aplicado al triángulo $ABC$, nos dice que \[\frac{a}{\mathrm{sen}(45)}=\frac{b}{\mathrm{sen}(105)}=\frac{c}{\mathrm{sen}(30)}\ \Longleftrightarrow\ \begin{cases}b^2=\frac{2+\sqrt{3}}{2}a^2\\c^2=\frac{1}{2}a^2.\end{cases}\] Aquí hemos usado un poco de trigonometría para calcular: \[\mathrm{sen}^2(105)=\cos^2(15)=\frac{1+\cos(30)}{2}=\frac{2+\sqrt{3}}{4}\] y hemos usado los cuadrados para no tener que calcular la raíz cuadrada en la fórmula del coseno del ángulo mitad. Los cuadrados también están motivados por el hecho de que aparecen en la conocida fórmula para la mediana: \[m^2=\frac{b^2+c^2}{2}-\frac{a^2}{4}=\left(\frac{\frac{2+\sqrt{3}}{2}+\frac{1}{2}}{2}-\frac{1}{4}\right)a^2=\frac{2+\sqrt{3}}{4}a^2.\] Con todo esto, tenemos la fórmula que buscamos, pues \[a^2b^2=\frac{2+\sqrt{3}}{2}a^4=4m^2c^2\ \Longleftrightarrow\ ab=2mc.\]

Para ver que el ángulo $x=\angle AMB$ es $45^\circ$, probaremos que su seno al cuadrado es $\frac{1}{2}$ (como $x\lt 90^\circ$, esto termina la demostración). Usando el teorema del seno en el triángulo $ABM$, tenemos que \begin{align*} \frac{c}{\mathrm{sen}(x)}=\frac{m}{\mathrm{sen}(105)}&\ \Longleftrightarrow\ \mathrm{sen}^2(x)=\frac{c^2\,\mathrm{sen}^2(105)}{m^2}=\frac{\frac{1}{2}\cdot\frac{2+\sqrt{3}}{4}}{\frac{2+\sqrt{3}}{4}}=\frac{1}{2}. \end{align*}

Si crees que el enunciado contiene un error o imprecisión o bien crees que la información sobre la procedencia del problema es incorrecta, puedes notificarlo usando los siguientes botones:
Informar de error en enunciado Informar de procedencia del problema
Problema 723
Cuatro bolas negras y cinco bolas blancas se colocan, en orden arbitrario, alrededor de una circunferencia. Si dos bolas consecutivas son del mismo color, se inserta una nueva bola negra entre ellas. En caso contrario, se inserta una nueva bola blanca. Se retiran las bolas negras y blancas previas a la inserción. Repitiendo el proceso, ¿es posible obtener nueve bolas blancas?
pistasolución 1info
Pista. ¿Qué ocurriría en el paso previo a que todas sean blancas?
Solución. Veamos que la respuesta es negativa por reducción al absurdo. Supongamos que tras un cierto número de pasos se llega a que todas sean blancas. En el paso previo todas las bolas tienen que tener alternadamente colores distintos, pero esto es imposible con un número impar de bolas.

Nota. Da igual el color de las bolas iniciales siempre que al menos una de ellas sea negra (si empezamos con todas blancas no hay nada que demostrar).

Si crees que el enunciado contiene un error o imprecisión o bien crees que la información sobre la procedencia del problema es incorrecta, puedes notificarlo usando los siguientes botones:
Informar de error en enunciado Informar de procedencia del problema
Problema 103
Encontrar todos los números naturales \(n\in\mathbb{N}\) tales que \(3^n+5^n\) es múltiplo de \(3^{n-1}+5^{n-1}\).
pistasolución 1info
Pista. Demostrar que, si esto ocurre, entonces \(3^n+5^n=4(3^{n-1}+5^{n-1})\).
Solución. Observemos en primer lugar que \[3(3^{n-1}+5^{n-1})=3^n+3\cdot 5^n<3^n+5^n<5\cdot 3^{n-1}+5^n=5(3^{n-1}+5^{n-1})\] luego, si \(3^n+5^n\) es múltiplo de \(3^{n-1}+5^{n-1}\), entonces tiene que ser \(3^n+5^n=4(3^{n-1}+5^{n-1})\). Ahora bien, esto nos lleva a que \(5^n-4\cdot 5^{n-1}=4\cdot 3^{n-1}-3^n\), es decir, \(3^{n-1}=5^{n-1}\), igualdad que sólo se tiene para \(n=1\). Deducimos que el único natural para el que se cumple es \(n=1\) (observemos que, en tal caso, \(3^n+5^n=8\) y \(3^{n-1}+5^{n-1}=2\)).
Si crees que el enunciado contiene un error o imprecisión o bien crees que la información sobre la procedencia del problema es incorrecta, puedes notificarlo usando los siguientes botones:
Informar de error en enunciado Informar de procedencia del problema

Sesión 3 —  Sábado 22 de enero de 2005 (mañana)

Problema 724
Sean $\alpha$ y $\beta$ las raíces del polinomio $P(x)=3x^2+3mx+m^2-1$, siendo $m$ un número real. Probar que $P(\alpha^3)=P(\beta^3)$.
pistasolución 1solución 2info
Pista. Calcula $P(\alpha^3)-P(\beta^3)$ sacando un factor común $\alpha^3-\beta^3$ en la expresión resultante.
Solución. Podemos calcular directamente \begin{align*}P(\alpha^3)-P(\beta^3)&=3(\alpha^6-\beta^6)+3m(\alpha^3-\beta^3)\\ &=3(\alpha^3+\beta^3)(\alpha^3-\beta^3)+3m(\alpha^3-\beta^3)\\ &=3(\alpha^3-\beta^3)(\alpha^3+\beta^3+m) \end{align*} Como $\alpha\neq\beta$, tendremos que probar que $\alpha^3+\beta^3=-m$ y habremos terminado. Esto es un cálculo estándar de polinomios simétricos de las raíces del polinomio: \begin{align*} \alpha^3+\beta^3&=(\alpha+\beta)^3-3\alpha^2\beta-3\alpha\beta^2=(\alpha+\beta)^3-3\alpha\beta(\alpha+\beta)\\ &=(-m)^3-3\cdot\tfrac{m^2-1}{3}\cdot (-m)=-m^3+(m^3-m)=-m, \end{align*} donde hemos usado que conocemos la suma $\alpha+\beta=-m$ y el producto $\alpha\beta=\frac{m^2-1}{3}$ a partir de los coeficientes de $P(x)$ (relaciones de Cardano-Vieta).
Solución. Trabajando con la raíz $\alpha$, tenemos que $3\alpha^3+3m\alpha^2+(m^2-1)\alpha=0$, luego podemos multiplicar esta igualdad por $\alpha$ y despejar $\alpha^3$ como \[\alpha^3=-m\alpha^2+\tfrac{1-m^2}{3}\alpha=-m(\tfrac{1-m^2}{3}-m\alpha)+\tfrac{1-m^2}{3}\alpha=\tfrac{1+2m^2}{3}\alpha+\tfrac{m(m^2-1)}{3}.\] De esta manera, podemos evaluar (omitimos los cálculos de la simplificación): \begin{align*} P(\alpha^3)&=P(\tfrac{1+2m^2}{3}\alpha+\tfrac{m(m^2-1)}{3})\\ &=3(\tfrac{1+2m^2}{3}\alpha+\tfrac{m(m^2-1)}{3})^2+3m(\tfrac{1+2m^2}{3}\alpha+\tfrac{m(m^2-1)}{3})+m^2-1\\ &=\tfrac{(1+2m^2)^2}{9}(3a^2+3am+m^2-1)-\tfrac{1}{9}(m^6-3m^4-3m^2+8). \end{align*} Usando de nuevo que $\alpha$ es raíz, tenemos que $P(\alpha^3)=-\tfrac{1}{9}(m^6-3m^4-3m^2+8)$, pero el mismo cálculo vale para $\beta$ luego $P(\alpha^3)=P(\beta^3)$.

Nota. Aunque la otra solución propuesta es más elegante, hay trucos que permiten manipular potencias de las raíces de un polinomio con cierta fluidez. En una prueba de olimpiada puede preferirse un cálculo más largo si estamos convencidos de que así saldrá la solución frente a buscar opciones más elegantes.

Si crees que el enunciado contiene un error o imprecisión o bien crees que la información sobre la procedencia del problema es incorrecta, puedes notificarlo usando los siguientes botones:
Informar de error en enunciado Informar de procedencia del problema
Problema 725
En el interior de un cuadrado $ABCD$ se construye el triángulo equilátero $ABE$. Sea $P$ el punto intersección de las rectas $AC$ y $BE$. Sea $F$ el punto simétrico de $P$ respecto de la recta $DC$. Se pide demostrar que:
  1. El triángulo $CEF$ es equilátero.
  2. El triángulo $DEF$ es rectángulo e isósceles.
  3. El triángulo $BDF$ es isósceles.
  4. El triángulo $PDF$ es equilátero.
pistasolución 1info
Pista. ¡Caza de ángulos!
Solución. El triángulo $ECB$ es isósceles ya que tiene dos de sus lados iguales a los lados del cuadrado $ABCD$. Como $\angle EBC=90-60=30$, tenemos que $\angle BEC=\angle ECB=75$ (para que los ángulos de este triángulo sumen $180$). Esto nos dice que $\angle DCE=90-75=15$, con lo que $\angle FCE=15+45=60$. Ahora bien, también tenemos que $\angle EPC=\angle APB=180-\angle PAB-\angle PBA=180-60-45=75$, luego $PCE$ es un triángulo isósceles. Tenemos así que $CP=EC$, pero también tenemos que $CP=CF$ por simetría, luego $CEF$ es equilátero por tener dos lados iguales que forman un ángulo igual a $60$. Hemos respondido así al apartado (a).

Que $CEF$ es equilátero implica que $EF=CE=DE$, ya que $CDE$ es claramente isósceles. Además, esto último nos dice que $\angle DEF=180-15-15=150$, con lo que $\angle DEF=150-60=90$. Tenemos entonces probado el apartado (b): $DEF$ es rectángulo e isósceles porque tiene dos lados iguales que forman un ángulo recto.

Ahora observamos que $\angle DEB=60+75=\angle FEB$, luego la recta $EB$ es la bisectriz interior del ángulo recto $\angle DEF$. Esta bisectriz es mediatriz del triángulo isósceles $DEF$, luego cualquier punto de $BE$ está a la misma distancia de $D$ que de $F$. En particular, el triángulo $BDF$ es isósceles y ya tenemos (c).

Tenemos que $PDF$ es isósceles por el mismo motivo, luego será suficiente ver que $\angle DFP=60$ y habremos terminado (hay realmente muchas formas de hacerlo). Como $\angle DBE=60-45=15$, tenemos que $\angle EBF=15$, luego también tenemos que $\angle FBC=90-45-15-15=15$. En otras palabras, $FB$ es bisectriz (y por tanto mediatriz) en el triángulo isósceles $CBE$, luego también lo es del triángulo equilátero $EFC$. Esto nos lleva a que $\angle EFB=\angle BFE=30$. Si trazamos ahora el segmento $PF$, que es paralelo a $BC$ por la simetría, tenemos que $\angle PFB=\angle CBF=15$ por ser ángulos internos alternos, luego $\angle DFP=45+15=60$ como queríamos probar.

imagen
Si crees que el enunciado contiene un error o imprecisión o bien crees que la información sobre la procedencia del problema es incorrecta, puedes notificarlo usando los siguientes botones:
Informar de error en enunciado Informar de procedencia del problema
Problema 726
Encontrar todas las funciones $f:\mathbb{R}\to\mathbb{R}$ tales que \[x^2f(x)+f(1-x)=2x-x^4,\] para cualquier número real $x\in\mathbb{R}$.
pistasolución 1info
Pista. Haz el cambio de variable $x\to 1-x$ para obtener otra ecuación que relacione $f(x)$ y $f(1-x)$. Resuelve el sistema lineal que obtienes.
Solución. Cambiando $x$ por $1-x$, tenemos la ecuación \[(1-x)^2f(1-x)+f(x)=2(1-x)-(1-x)^4.\] Esta ecuación es cierta para todo $x\in\mathbb{R}$ y, junto con la ecuación dada en el enunciado, nos da un sistema lineal de dos ecuaciones si vemos a $f(x)$ y $f(1-x)$ como incógnitas. Restando a esta última ecuación la primera multiplicada por $(1-x)^2$ eliminamos el término $f(1-x)$, lo que nos da \[f(x)-x^2(1-x)^2f(x)=2(1-x)-(1-x)^4-(1-x)^2(2x-x^4).\] Tras simplificar y factorizar (observamos que $1-x^2(1-x)^2$ es diferencia de cuadrados), podemos despejar \[f(x)=\frac{-(x-1)(x+1)(x^2-x-1)(x^2-x+1)}{(x^2-x-1)(x^2-x+1)}.\] El factor $x^2-x+1$ es siempre positivo ya que la ecuación $x^2-x+1=0$ no tiene soluciones. Sin embargo $x^2-x-1=0$ tiene soluciones $x=\frac{1}{2}(1\pm\sqrt{5})$ en cuyo caso no podemos simplificar este factor. Distinguimos dos casos:
  • Si $x\neq\frac{1}{2}(1\pm\sqrt{5})$, entonces tras simplificar todos los factores comunes obtenemos que $f(x)=1-x^2$.
  • Para el caso $x=\frac{1}{2}(1\pm\sqrt{5})$, sustituimos estos valores en la ecuación inicial, lo que nos da el siguiente sistema lineal en las incógnitas $a=\frac{1}{2}(1-\sqrt{5})$ y $b=\frac{1}{2}(1-\sqrt{5})$: \[\left\{\begin{array}{l}2a+(3+\sqrt{5})b=-5-\sqrt{5},\\ (3-\sqrt{5})a+2b=-5+\sqrt{5}\end{array}\right.\] Es fácil ver que es compatible indeterminado y que podemos despejar \[b=\frac{-5+\sqrt{5}-(3-\sqrt{5})a}{2},\] de forma que $a$ es un parámetro real arbitrario.
Con todo esto, las funciones que verifican la igualdad del enunciado son las de la forma \[f(x)=\begin{cases}1-x^2&\text{si }x\neq\frac{1}{2}(1\pm\sqrt{5}),\\ a&\text{si }x=\frac{1}{2}(1-\sqrt{5}),\\ \frac{-5+\sqrt{5}-(3-\sqrt{5})a}{2}&\text{si }x=\frac{1}{2}(1+\sqrt{5}),\end{cases}\] para cualquier $a\in\mathbb{R}$.

Nota. Este problema se ha marcado con 2,5 estrellas porque es realmente fácil omitir el caso $x=\frac{1}{2}(1\pm\sqrt{5})$ y decir que $f(x)=1-x^2$ para todo $x\in\mathbb{R}$ (este fallo está incluso en la solución oficial), que se corresponde con el caso $a=\frac{-1+\sqrt{5}}{2}$.

Si crees que el enunciado contiene un error o imprecisión o bien crees que la información sobre la procedencia del problema es incorrecta, puedes notificarlo usando los siguientes botones:
Informar de error en enunciado Informar de procedencia del problema

Sesión 4 —  Sábado 22 de enero de 2005 (tarde)

Problema 727
Un grupo de chicos y chicas han comido en un restaurante en el que solo se sirven pizzas cortadas en $12$ raciones. Cada chico quería comer $6$ o $7$ raciones y cada chica $2$ o $3$ raciones. Al pedir, vieron que $4$ pizzas no eran suficientes y que con $5$ pizzas había de sobra. Calcular el número de chicos y de chicas del grupo.
pistasolución 1info
Pista. Expresa con desigualdades que el mínimo que querían comer excede las 48 porciones dadas por 4 pizzas, mientras que el máximo que podían comer no supera los 60 trozos. Resuelve el sistema de inecuaciones resultante.
Solución. Pongamos que hay $a$ chicos y $b$ chicas. Si todos hubieran comido el mínimo, habrían hecho falta $6a+2b$ porciones, mientras que el máximo sería $7a+3b$. Por tanto, se tiene que cumplir que \[48\lt 6a+2b,\qquad 7a+3b\lt 60.\] Observamos que las desigualdades son estrictas porque se dice que faltó con cuatro pizzas y sobró con cinco. Sumando siete veces la primera desigualdad y seis la segunda, tenemos que $336+42a+18b\lt 42a+14b+360$, de donde $b\lt 6$, es decir $b\leq 5$. Distingamos casos:
  • Si $b=1$, las desigualdades quedan $46\lt 6a$ y $7a\lt 57$, es decir, $\frac{46}{6}\lt a\lt\frac{57}{7}$ que tiene por única solución entera $a=8$.
  • Si $b=2$, queda $44\lt 6a$ y $7a\lt 54$, que no tiene soluciones enteras.
  • Si $b=3$, queda $42\lt 6a$ y $7a\lt 51$, que no tiene soluciones enteras.
  • Si $b=4$, queda $40\lt 6a$ y $7a\lt 48$, que no tiene soluciones enteras.
  • Si $b=5$, queda $38\lt 6a$ y $7a\lt 45$, que no tiene soluciones enteras.
Deducimos que hay dos soluciones al problema: 8 chicos y 1 chica o bien 7 chicos y 2 chicas.
Si crees que el enunciado contiene un error o imprecisión o bien crees que la información sobre la procedencia del problema es incorrecta, puedes notificarlo usando los siguientes botones:
Informar de error en enunciado Informar de procedencia del problema
Problema 728
Demostrar hay infinitas ternas de números enteros $(x,y,z)$ verificando la siguiente ecuación: \[x^2+y^2-z^2-x-3y-z-4=0.\]
pistasolución 1info
Pista. Completa cuadrados en cada variable para eliminar los términos lineales y observa que te quedan muchos cuadrados.
Solución. Podemos completar los cuadrados en cada incógnita para expresar de forma equivalente la ecuación como \[(x-\tfrac{1}{2})^2-\tfrac{1}{4}+(y-\frac{3}{2})^2-\tfrac{9}{4}-(z+\tfrac{1}{2})^2+\tfrac{1}{4}=4.\] Tras pasar los términos independientes al miembro de la derecha y multiplicar por 4, tenemos la ecuación equivalente \[(2x-1)^2+(2y-3)^2-(2z+1)^2=25.\] Por tanto, tomando $z=x-1$ e $y=4$, tenemos la familia infinita de soluciones $(x,y,z)=(a,4,a-1)$ para cualquier entero $a$.
Si crees que el enunciado contiene un error o imprecisión o bien crees que la información sobre la procedencia del problema es incorrecta, puedes notificarlo usando los siguientes botones:
Informar de error en enunciado Informar de procedencia del problema
Problema 729
En un tablero de ajedrez $10\times 10$ se colocan $41$ torres. Probar que se pueden elegir al menos $5$ de ellas que no se ataquen entre sí.
pistasolución 1info
Pista. Si divides el tablero en 10 subconjuntos (de 10 casillas cada uno), entonces el principio del palomar te asegura que alguno de esos conjuntos tendrá cinco torres.
Solución. Numeramos las casillas del tablero del $0$ al $9$ siguiendo las diagonales: \[\begin{bmatrix} 0&1&2&3&4&5&6&7&8&9\\ 1&2&3&4&5&6&7&8&9&0\\ 2&3&4&5&6&7&8&9&0&1\\ 3&4&5&6&7&8&9&0&1&2\\ 4&5&6&7&8&9&0&1&2&3\\ 5&6&7&8&9&0&1&2&3&4\\ 6&7&8&9&0&1&2&3&4&5\\ 7&8&9&0&1&2&3&4&5&6\\ 8&9&0&1&2&3&4&5&6&7\\ 9&0&1&2&3&4&5&6&7&8 \end{bmatrix}\] Al colocar $41$ torres, por el principio del palomar al menos cinco de ellas compartirán el número que hay en su casilla. Estas torres no se atacan entre sí ya que ningún número se repite en ninguna fila ni columna.
Si crees que el enunciado contiene un error o imprecisión o bien crees que la información sobre la procedencia del problema es incorrecta, puedes notificarlo usando los siguientes botones:
Informar de error en enunciado Informar de procedencia del problema
José Miguel Manzano © 2010-2024. Esta página ha sido creada mediante software libre